During the recent economic downturn, banks contributed to the decline by loaning less money. Prior to the downturn, ...

Shiyi-Zhang on March 3, 2019

Why is B incorrect?

Why is B incorrect?

Replies
Create a free account to read and take part in forum discussions.

Already have an account? log in

Ravi on March 9, 2019

@Shiyi-Zhang,

Happy to help!

In the stimulus, the author is saying that before the economic
downturn, regulatory standards for loan-making by banks were tightened
and that during the downturn, banks contributed to the decline by
loaning less money. The author concludes from these premises that if
standards are relaxed, banks will lend more money.

The question says, "The argument assumes that..."

This is a strengthen with a necessary premise question. We're looking
for something that must be true in order for the argument to stand a
chance and that, if false, wrecks the argument.

In the stimulus, there is a big gap between the authors' premises and
conclusion. We don't know for sure that because the regulations on
banks came before the reduction in loans that the regulations caused
the reduction in loans. The author is implying causation from a
correlation. In doing so, the author is assuming that regulations are
the sole cause of banks lending less. Let's take a look at the
answers.

You asked about (B).

(B) says, "the imposition of the tighter regulatory standards was not
a cause of the economic downturn"

The problem with (B) is that it's confusing what the author is saying
is causing what. The author's conclusion is that the increase in bank
regulations caused the reduction in loans. The cause of the economic
downtown is irrelevant for our considerations since the author's
conclusion doesn't have to do with the economic downturn. As a result,
we can get rid of this choice.

(A) says, "the downturn did not cause a significant decrease in the
total amount of money on deposit with banks which is the source of
funds for banks to lend"

(A)'s negation is the downturn did cause a significant decrease in the
total amount of money on deposit with banks which is the source of
funds for the banks to lend. If (A)'s negation were true, then the
argument would fall apart because there's an alternative cause for the
decrease in loans. Since (A) is eliminating an alternative cause for
the reduction in loans, it is a necessary premise for the argument and
is the correct answer choice.

Does this make sense? Let us know if you have any more questions!

Anna20 on July 19, 2020

Why is this not a strengthen with sufficient premise question? It was my understanding that here the answer had to bridge the gap between the premise & conclusion.